Bibm@th

Forum de mathématiques - Bibm@th.net

Bienvenue dans les forums du site BibM@th, des forums où on dit Bonjour (Bonsoir), Merci, S'il vous plaît...

Vous n'êtes pas identifié(e).

Répondre

Veuillez composer votre message et l'envoyer
Nom (obligatoire)

E-mail (obligatoire)

Message (obligatoire)

Programme anti-spam : Afin de lutter contre le spam, nous vous demandons de bien vouloir répondre à la question suivante. Après inscription sur le site, vous n'aurez plus à répondre à ces questions.

Quel est le résultat de l'opération suivante (donner le résultat en chiffres)?
vingt cinq moins quatorze
Système anti-bot

Faites glisser le curseur de gauche à droite pour activer le bouton de confirmation.

Attention : Vous devez activer Javascript dans votre navigateur pour utiliser le système anti-bot.

Retour

Résumé de la discussion (messages les plus récents en premier)

Black Jack
23-01-2021 08:16:24

Bonjour,

Démontrer l'inégalité arithmético-géométrique pour un nombre n quelconque de nombres n'est sûrement pas niveau Collège, par contre se limiter à ce qui est nécessaire pour répondre au problème posé initialement, comme fait dans les messages 29 et 32 devrait probablement l'être.

Bernard-maths
22-01-2021 17:45:00

Bonsoir à tous !

je suis toujours à distance. Mais la récurrence a l'air de fonctionner ! Bravo !

Mais moi je dis, ce n'est pas du collège ! Du lycée avec des (très) bons élèves, oui ...

Qui est vraiment Nicolas Tesla, l'invité ???

En tout cas, il a du magnétisme ... puisqu'on a suivi.

Bonne soirée, Bernard-maths

yoshi
21-01-2021 15:15:55

Re,

J'ai trouvé ça, ce matin (sans appel à Jensen et la convexité) :
https://maths-olympiques.fr/wp-content/ … s-Theo.pdf
Je suis en train de l'éplucher et je l'ai déjà récrit en partie, parce que la présentation d"origine est un peu "dense" :
(On a déjà vu que ça marchait pour 2 et 4)

.
Supposons l’inégalité et le cas d’égalité prouvé au rang n.
Soient $x_1, \cdots,x_{n+1}$ des réels positifs.
Notons
$S=x_1+···+x_n+x_{n+1}$.
Comme S est compris entre le minimum des $x_i$ et le maximum des $x_i$, quitte à renuméroter les $x_i$, on peut supposer $x_1$ étant le minimum des $x_i$, $x_2$ le maximum.
Posons $y1=S$, $y_2=x_2− (S−x_1) = x_2−S+x_1\geqslant 0$,  et $y_i=x_i$ pour $3\leqslant i\leqslant n+1$,  les $y_i$ sont  bien  tous  positifs. 
On  a $y_2+···+y_{n+1}= x_1+···x_{n+1}−S= (n+1)S−S=nS$.
Ce qui signifie que $\dfrac{y_2+···+y_n+1}{n}=S$.

D’après l’inégalité arithmético-géométrique, on a
$y_2+···+y_{n+1}\geqslant \left(\dfrac{y_2+···+y_{n+1}}{n}\right)^n=S^n$.

En particulier :
$y_1\times y_2\cdots \times y_{n+1}\leqslant S^{n+1}$.
Or
$y_1y_2−x_1x_2=S(x_1+x_2−S) −x_1x_2=Sx_1+Sx_2−S^2−x_1x_2= (S−x_1)(x_2−S)\geqslant 0$
Donc
$x_1x_2\leqslant y_1y_2$.
En particulier :
$x_1. . .x_{n+1}\leqslant y_1. . .y-{n+1}=S_{n+1}$,
donc
$(n+1)\sqrt[n+1]{x_1. . .x_{n+1}}\leqslant S=\dfrac{x_1+···+x_{n+1}}{n+1}$
L’inégalité est prouvée.

Il y a encore :

Pour le cas d’égalité, supposons qu’on ait égalité.
Notons d’abord que si un des $x_i$ est nul, on a égalité si et seulement si $\dfrac {x_1+...x_{n+1}}{n+1}=0$.
Comme les $x_i$ sont positifs, pour avoir égalité, il faut que tous les $x_i$ soient nuls, donc $x_1=···=x{n+1}$.
Si les $x_i$ sont tous non nuls, en particulier, S est non nul car supérieur au minimum des $x_i$, et les $yi$ sont non nuls (c’est évident pour $i\geqslant 3$, pour $i=1$, ça vient du fait que $S >0$, et  $y_2=x_2−S+x_1>x1>0$).
En particulier, si on a égalité, on a égalité dans les deux inégalités précédentes :
c’est à dire dans $y_2+···+y_{n+1}\leqslant  \left(\dfrac{y_2+···+y_{n+1}}{n}\right)^n$
et
$y_1y_2−x_1x_2=S(x_1+x_2−S)−x_1x_2=Sx_1+Sx_2−S^2−x_1x_2= (S−x_1)(x_2−S)\geqslant 0$.
Par hypothèse de récurrence, cela donne que $y_2=y_3=···=y_{n+1}$ et $x_1=S$ ou $x_2=S$.
En particulier $x_3=···=x_{n+1}$.
Si $x_1=S$, alors $y_2=x_2=x_3 \cdots  x_{n+1}$, ainsi $S=\dfrac {S+x_2+...x_{n+1}}{n+1}=\dfrac{S+n x_2}{n+1}$.
On obtient que $(n+1)S=S+nx_2$, donc $nx_2=nS$ donc $x_2=S$. En particulier $S=x_1=x_2=···=x_{n+1}$.
On obtient le même résultat si $x_2=S$.
Réciproquement
si $x1=···=x_{n+1}$, $\dfrac{x_1+···+x_{n+1}}{n+1}=x_1=(n+1)\sqrt[n+1]{x1. . .x_{n+1}}$

On a bien égalité...

Je ne crois pas avoir commis d'erreurs de transcription...

@+

Roro
21-01-2021 14:28:35
Black Jack a écrit :

Roro, ici on a besoin que du cas à 4 nombres et la démo est alors sans difficulté.

Super Black Jack : ça m'a l'air de bien fonctionner même au niveau collège !

Pour répondre à Bernard-maths, oui c'est généralisable à $n$ variables, c'est simplement l'inégalité arithmético-géométrique.

Roro.

Bernard-maths
21-01-2021 13:48:18

Bonjour à tous !

Les dernières réponses sont très sympas, merci.

Mais je me demandais si la formule est extensible à n fractions et n facteurs ?

Comme c'est vrai pour a/b + b/a, je pensais à un raisonnement par récurrence ...

Mais je n'ai pas vraiment cherché ! Je suis "trop" pris par mes polyèdres ...

Si quelqu'un y pense, je suis les débats !

Cordialement, Bernard-maths.

Black Jack
21-01-2021 10:11:37
Roro a écrit :

Bonsoir,

J'avais évoqué cette façon de faire dans le post #13 :

Roro a écrit :

Si on veut généraliser, on se trouve assez rapidement à utiliser de l'analyse convexe (niveau sup.) pour montrer l'inégalité arithmético-géométrique (qui est la généralisation avec $n$ variables).
Roro.

Mais en fait, c'est justement cette inégalité qui n'est pas "triviale" et toute la difficulté réside dans la preuve de ce truc (qu'on sait faire en utilisant de la convexité du logarithme... ce qui n'est en soit peut être pas si difficile, surtout si on admet le lien entre convexité et dérivée seconde, etc.)! Finalement, la vraie question sera : qu'admet-on ? Evidemment si on admet l'inégalité arithmético-géométrique, c'est moins difficile puisque c'est plus ou moins la question.

Roro.

Bonjour,

Roro, ici on a besoin que du cas à 4 nombres et la démo est alors sans difficulté.

Soit A, B, C et D 4 nombres réels positifs.

[tex](\sqrt{A} - \sqrt{B})^2 = A + B - 2\sqrt{AB} \geq 0 [/tex] puisque un carré est toujours >= 0

[tex] A + B \geq 2\sqrt{AB}[/tex]
[tex] \frac{A + B}{2} \geq \sqrt{AB}[/tex]

De manière analogue, on a : [tex] \frac{C + D}{2} \geq \sqrt{CD}[/tex]
****
On a donc aussi :

[tex] \frac{\frac{A + B}{2} + \frac{C + D}{2}}{2} \geq \sqrt{\frac{A + B}{2} * \frac{C + D}{2}} [/tex]

[tex] \frac{\frac{A + B}{2} + \frac{C + D}{2}}{2}  \geq \sqrt{\sqrt{AB} * \sqrt{CD}} [/tex]

[tex] \frac{A+B+C+D}{4} \geq \sqrt[4]{ABCD}[/tex]

Roro
20-01-2021 22:13:59

Bonsoir,

J'avais évoqué cette façon de faire dans le post #13 :

Roro a écrit :

Si on veut généraliser, on se trouve assez rapidement à utiliser de l'analyse convexe (niveau sup.) pour montrer l'inégalité arithmético-géométrique (qui est la généralisation avec $n$ variables).
Roro.

Mais en fait, c'est justement cette inégalité qui n'est pas "triviale" et toute la difficulté réside dans la preuve de ce truc (qu'on sait faire en utilisant de la convexité du logarithme... ce qui n'est en soit peut être pas si difficile, surtout si on admet le lien entre convexité et dérivée seconde, etc.)! Finalement, la vraie question sera : qu'admet-on ? Evidemment si on admet l'inégalité arithmético-géométrique, c'est moins difficile puisque c'est plus ou moins la question.

Roro.

yoshi
20-01-2021 14:32:44

Re,

Ah oui !
Suffisamment "simple" pour être la réponse définitive à ce niveau...
Je le récris via Latex.
$\dfrac{A+B+C+D}{4}\geqslant \sqrt[4]{A\times B\times C\times D}$
$\Leftrightarrow$

$A+B+C+D \geqslant 4\sqrt[4]{A\times B\times C\times D}$
Avec
$A=\dfrac a b,\;B=\dfrac b c,\;C=\dfrac  c d \text{ et }D=\dfrac  d a$

on a alors :

$\dfrac a b+\dfrac b c+\dfrac  c d +\dfrac  d a \geqslant 4 \sqrt[4]{\dfrac a b\times\dfrac b c\times\dfrac  c d \times\dfrac  d a}$
$\Leftrightarrow$
$\dfrac a b+\dfrac b c+\dfrac  c d +\dfrac  d a \geqslant 4\sqrt[4] 1$
$\Leftrightarrow$
$\dfrac a b+\dfrac b c+\dfrac  c d +\dfrac  d a \geqslant  4$

Je ne vois pas de faille (je ne vois d'ailleurs pas où elle pourrait être).

Bravo.

@+

Black Jack
20-01-2021 12:58:45

Bonjour,

Et ainsi ?

moyenne arithmétique >= moyenne géométrique (propriété connue ou à démonter si non) --->

(A+B+C+D)/4 >= (ABCD)^(1/4)

avec A = a/b ; B = b/c ; C = c/d ; D = d/a, il vient :

a/b + b/c + c/d + d/a >= 4 * (1)^(1/4)

a/b + b/c + c/d + d/a >= 4

Roro
19-01-2021 23:36:55

Bonsoir,

Je peux essayer de faire le point et résumer :

Le problème : Montrer que pour tous $a$, $b$, $c$ et $d$, réels strictement positifs, on a $\displaystyle \frac{a}{b}+\frac{b}{c} + \frac{c}{d} + \frac{d}{a} \geq 4$.

Il y a eu deux méthodes proposées, qui amènent toutes les deux au résultat mais utilisent des notions "niveau Supérieur" :

Méthode 1

Si on introduit la fonction de 4 variables définie par $f(a,b,c,d) = \displaystyle \frac{a}{b}+\frac{b}{c} + \frac{c}{d} + \frac{d}{a}$, on remarque que pour tout $\lambda>0$, on a $f(\lambda a,\lambda b,\lambda c,\lambda d) = f(a,b,c,d)$. Autrement dit, on peut choisir $a=1$ et considérer la fonction de 3 variables définie par $g(b,c,d) = \displaystyle \frac{1}{b}+\frac{b}{c} + \frac{c}{d} + d$.

En calculant le gradient de $g$, on montre facilement qu'il s'annule uniquement pour $b=c=d=1$.

Il faut ensuite vérifier que cet extremum est un minimum. Pour cela on peut calculer la Hessienne en $(1,1,1)$ : il s'agit de l'application bilinéaire dont la forme quadratique associée est définie par
$$Q(b,c,d) = \partial^2_{bb} g(1,1,1)\frac{b²}{2} +  \partial^2_{cc} g(1,1,1)\frac{c²}{2} +\partial^2_{dd} g(1,1,1)\frac{d²}{2} + \partial^2_{bc} g(1,1,1) bc + \partial^2_{bd} g(1,1,1) bd + + \partial^2_{cd} g(1,1,1) cd.$$

Un calcul des dérivées secondes montre que $Q(b,c,d) =b²+c²+d²-bc-cd$. Cette forme quadratique est définie positive (utiliser par exemple la méthode de Gauss pour vérifier que sa signature est $(3,0)$) de sorte que le point $(1,1,1)$ est bien un minimum.

Finalement, on en déduit que pour tous $a$, $b$, $c$ et $d$, réels strictement positifs, on a $f(a,b,c,d) \geq f(1,1,1,1) = 4$.

Méthode 2

On suppose d'abord qu'il existe quatre réels strictement positifs $a$, $b$, $c$, $d$ pour lesquels on atteint le minimum et on note $m$ ce minimum : $m=\frac{a}{b}+\frac{b}{c}+\frac{c}{d}+\frac{d}{a}$. Comme dans le post #11, on en déduit que ce minimum est nécessairement supérieur à $4$. Comme on sait que $4$ est atteint (en  prenant par exemple $a=b=c=d=1$) cela terminera la preuve.

Reste à montrer que le minimum est bien atteint.

Un peu comme pour la méthode 1, j'introduis la fonction de 3 variables définie par $g(b,c,d) = \displaystyle \frac{1}{b}+\frac{b}{c} + \frac{c}{d} + d$.

- On remarque que pour $b<\frac{1}{4}$ on a $g(b,c,d)>\frac{1}{b}>4$. Puisqu'on sait que le minimum recherché est inférieur où égal à $4$ (puisque la valeur $4$ est atteinte), s'il est atteint c'est forcément parmi les valeurs $(b,c,d)$ avec $b\geq 1/4$. On se concentre donc à partir de maintenant sur les cas où $b\geq 1/4$.

- Pour $c<\frac{1}{16}$, on a alors $g(b,c,d)>\frac{b}{c} > \frac{16}{4} = 4$. Comme précédemment, on se concentre sur les cas où $c\geq 1/16$.

- Pour $d<\frac{1}{64}$, on a $g(b,c,d)>\frac{c}{d} > \frac{64}{16} =4$. Comme précédemment, on se concentre sur les cas où $d\geq 1/64$.

- Pour $d>4$, on a $g(b,c,d)>d>4$, on se concentre sur les cas où $d \leq 4$.

- Pour $c>16$, on a $g(b,c,d)>\frac{c}{d}>\frac{16}{4} = 4$, on se concentre sur les cas où $c \leq 16$.

- Pour $b>64$, on a $g(b,c,d)>\frac{b}{c}>\frac{64}{16} = 4$, on se concentre sur les cas où $b \leq 64$.

Finalement, si le minimum de $g$ est atteint, c'est forcément sur le compact $\displaystyle [\frac{1}{4}, 64]\times [\frac{1}{16}, 16] \times [\frac{1}{64}, 4]$.

La fonction $g$ étant continue sur ce compact, le minimum est atteint.

Roro.

Zebulor
19-01-2021 21:03:33

Bonsoir,
@Bernard : en fait je crois comprendre que DM veut dire : devoir maison !

Bernard-maths
19-01-2021 20:57:19

Bonsoir à tous !

Je suis un peu embrouillé par tous vos essais, désolé ! Je crois que Nicolas Tesla n'est plus au collège, et que son DM veut dire "Défi Matheux" ?

Je prend 4 réels au hasard, et je leur donne un nom : il y a 4*3*2*1 = 4 ! = 24 façons de le faire.

Si je les range en ordre décroissant, a>b>c>d, j'obtiens des rapports a/b, b/c, c/d et d/a "minimaux", et si j'additionne 4 rapports minimaux ... je dois obtenir une somme minimale, parmi les 24 possibilités ... à vérifier ! Mais on doit pouvoir le faire en choisissant pour chaque rapport le 2ème nombre à prendre ...

Alors les 3 premiers rapports sont > 1, et le dernier < 1, donc ça fait au moins 3, plus des broutilles.

Est-ce que broutilles > ou = à 1 ?

J'ai essayé de poser a/b = 1 +eps1, b/c = 1+esp2, c/d = 1+eps3. Que vaut d/ a ?
Comme a/b * b/c * c/d * d/a = 1, alors d/a = ... = 1/[(1+eps1) (1+eps2) (1+eps3)] = 1/[1+eps1+eps2+eps3+etc...] = 1/[1 + X].

Et comme 1/(1+X) = 1 - X + X² - X^3 + X^4 - ... ] un coup par défaut, un coup par excès ...

On se retrouve avec a/b + b/c + c/d + d/a  = 4 - ou + ... selon le développement ...

Et pour le moment, je cale dans les calculs, avec des eps^n !

Il me semble que certains ont essayé quelque chose de semblable ?

Mais c'est amusant !

Bonne soirée, Bernard-maths

Zebulor
19-01-2021 09:19:21
Roro a écrit :

Mais je viens de dire à mon post précédent que le minimum ($0$) n'est pas atteint !!!

Dont acte... mais je ne vois pas bien le rapport avec mon dernier post

Roro
19-01-2021 09:09:25
Zebulor a écrit :

Re,
serais-je mal reveillé ... je ne l'exclus pas après tout..
alors supposons que le miminum soit atteint (c'est très probablement faux) et qu'il soit m=$\frac{1}{2}+\frac{2}{3}+\frac{3}{4}$.
On est bien d'accord que $A \le m$ ?

Mais je viens de dire à mon post précédent que le minimum ($0$) n'est pas atteint !!!

Chlore au quinoa a écrit :

Dommage pour le minimum, j'essaie de voir si c'est possible de prouver qu'il y en a un sans invoquer de différentiabilité, je reviens vers vous si je trouve !

Pour prouver que le minimum est atteint, il n'y a à mon avis pas besoin de différentiabilité. Un argument de compacité (topologique) suffit... mais c'est pas simple à expliquer au niveau du lycée. Je persiste à croire qu'il y a une façon astucieuse de faire apparaître des carrés.

Roro.

Zebulor
19-01-2021 09:05:30

Re,
serais-je mal reveillé ... je ne l'exclus pas après tout..
alors supposons que le miminum soit atteint (c'est très probablement faux) et qu'il soit m=$\frac{1}{2}+\frac{2}{3}+\frac{3}{4}$.
On est bien d'accord que $A \lt m$ ?

Pied de page des forums